HideoSatou's blog

By HideoSatou, history, 4 years ago, In English

Given a planar directed acyclic graph with $$$N$$$ vertices and $$$M$$$ edges. Given $$$Q$$$ queries of type $$$(a, b)$$$. For each query, output "Yes" if it's possible to reach vertex $$$b$$$ from vertex $$$a$$$ and "No" otherwise.

$$$1 \le N, M, Q \le 300000$$$.

Is it possible to solve this problem under this constraints?

Full text and comments »

  • Vote: I like it
  • +24
  • Vote: I do not like it